Respiratory

Ace your homework & exams now with Quizwiz!

A frail 82-year-old female patient develops sudden shortness of breath while sitting in a chair. What location on the chest should the nurse begin auscultation of the lung fields? A. Bases of the posterior chest area B. Apices of the posterior lung fields C. Anterior chest area above the breasts D. Midaxillary on the left side of the chest

A Rationale: Baseline data with the most information is best obtained by auscultation of the posterior chest, especially in female patients because of breast tissue interfering with the assessment or if the patient may tire easily (e.g., shortness of breath, dyspnea, weakness, fatigue). Usually auscultation proceeds from the lung apices to the bases unless it is possible the patient will tire easily. In this case the nurse should start at the bases.

The patient is calling the clinic with a cough. What assessment should be made first before the nurse advises the patient? A Cough sound, sputum production, pattern B Frequency, a family history, hematemesis C Smoking, medications, residence location D Weight loss, activity tolerance, orthopnea

A Rationale: The sound of the cough, sputum production and description, as well as pattern of the cough's occurrence (including acute or chronic) and what its occurrence is related to are the first assessments to be made to determine the severity. Frequency of the cough will not provide a lot of information. Family history can help to determine a genetic cause of the cough. Hematemesis is vomiting blood and not as important as hemoptysis. Smoking is an important risk factor for COPD and lung cancer and may cause a cough. Medications may or may not contribute to a cough as does residence location. Weight loss, activity intolerance, and orthopnea may be related to respiratory or cardiac problems, but are not as important when dealing with a cough.

During the assessment in the ED, the nurse is palpating the patient's chest. Which finding is a medical emergency? A Trachea moved to the left B Increased tactile fremitus C Decreased tactile fremitus D Diminished chest movement

A Rationale: Tracheal deviation is a medical emergency when it is caused by a tension pneumothorax. Tactile fremitus increases with pneumonia or pulmonary edema and decreases in pleural effusion or lung hyperinflation. Diminished chest movement occurs with barrel chest, restrictive disease, and neuromuscular disease.

A nurse teaches a client who has chronic obstructive pulmonary disease. Which statements related to nutrition should the nurse include in this client's teaching? (Select all that apply.) a. "Avoid drinking fluids just before and during meals." b. "Rest before meals if you have dyspnea." c. "Have about six small meals a day." d. "Eat high-fiber foods to promote gastric emptying." e. "Increase carbohydrate intake for energy."

A, B, C

A nurse plans care for a client who has chronic obstructive pulmonary disease and thick, tenacious secretions. Which interventions should the nurse include in this client's plan of care? (Select all that apply.) a. Ask the client to drink 2 liters of fluids daily. b. Add humidity to the prescribed oxygen. c. Suction the client every 2 to 3 hours. d. Use a vibrating positive expiratory pressure device. e. Encourage diaphragmatic breathing.

A, B, D

When admitting a 45-year-old female with a diagnosis of pulmonary embolism, the nurse will assess the patient for which of the following risk factors (select all that apply)? A) Obesity B) Pneumonia C) Hypertension D) Cigarette smoking E) Recent long distance travel

A, C, D, E

40. The nurse notes that a patient has incisional pain, a poor cough effort, and scattered rhonchi after a thoracotomy. Which action should the nurse take first? a. Assist the patient to sit upright in a chair. b. Splint the patient's chest during coughing. c. Medicate the patient with prescribed morphine. d. Observe the patient use the incentive spirometer.

ANS: C A major reason for atelectasis and poor airway clearance in patients after chest surgery is incisional pain (which increases with deep breathing and coughing). The first action by the nurse should be to medicate the patient to minimize incisional pain. The other actions are all appropriate ways to improve airway clearance but should be done after the morphine is given.

The nurse evaluates that discharge teaching for a patient hospitalized with pneumonia has been effective when the patient makes which statement about measures to prevent a relapse? A. "I will seek immediate medical treatment for any upper respiratory infections." B. "I should continue to do deep-breathing and coughing exercises for at least 12 weeks." C. "I will increase my food intake to 2400 calories a day to keep my immune system well." D. "I must have a follow-up chest x-ray in 6 to 8 weeks to evaluate the pneumonia's resolution."

Ans: D The follow-up chest x-ray will be done in 6 to 8 weeks to evaluate pneumonia resolution. A patient should seek medical treatment for upper respiratory infections that persist for more than 7 days. It may be important for the patient to continue with coughing and deep breathing exercises for 6 to 8 weeks, not 12 weeks, until all of the infection has cleared from the lungs. Increased fluid intake, not caloric intake, is required to liquefy secretions.

The patient seeks relief from the symptoms of an upper respiratory infection (URI) that has lasted for 5 days. Which patient assessment should the nurse use to help determine if the URI has developed into acute sinusitis? A. Coughing B. Fever, chills C. Dust allergy D. Maxillary pain

Ans: D The nurse should assess the patient for sinus pain or pressure as a clinical indicator of acute sinusitis. Coughing and fever are nonspecific clinical indicators of a URI. A history of an allergy that is likely to affect the upper respiratory tract is supportive of the sinusitis diagnosis but is not specific for sinusitis.

After admitting a patient to the medical unit with a diagnosis of pneumonia, the nurse will verify that which of the following physician orders have been completed before administering a dose of cefotetan (Cefotan) to the patient? A) Orthostatic blood pressures B) Sputum culture and sensitivity C) Pulmonary function evaluation D) Serum laboratory studies ordered for am

B

When caring for a patient who is 3 hours postoperative laryngectomy, the nurse's highest priority assessment would be A) Patient comfort. B) Airway patency. C) Incisional drainage. D) Blood pressure and heart rate.

B

The nurse is caring for a patient with chronic obstructive pulmonary disorder (COPD) and pneumonia who has an order for arterial blood gases to be drawn. What is the minimum length of time the nurse should plan to hold pressure on the puncture site? A 2 minutes B 5 minutes C 10 minutes D 15 minutes

B Rationale: After obtaining blood for an arterial blood gas measurement, the nurse should hold pressure on the puncture site for 5 minutes by the clock to be sure that bleeding has stopped. An artery is an elastic vessel under much higher pressure than veins, and significant blood loss or hematoma formation could occur if the time is insufficient.

A nurse assesses a client who has a chest tube. For which manifestations should the nurse immediately intervene? (Select all that apply.) a. Production of pink sputum b. Tracheal deviation c. Sudden onset of shortness of breath d. Pain at insertion site e. Drainage of 75 mL/hr

B, C

A nurse assesses a client with chronic obstructive pulmonary disease. Which questions should the nurse ask to determine the client's activity tolerance? (Select all that apply.) a. "What color is your sputum?" b. "Do you have any difficulty sleeping?" c. "How long does it take to perform your morning routine?" d. "Do you walk upstairs every day?" e. "Have you lost any weight lately?"

B, C, E

laryngectomy

Immediately after surgery, the patient with a laryngectomy requires frequent suctioning by means of the laryngectomy tube. Secretions typically change in amount and consistency over time. The patient may initially have copious blood-tinged secretions that diminish and thicken. Normal saline bolus through the tracheostomy tube is not recommended to assist with removal of thickened secretions because it causes hypoxia and damage to the epithelial cells.

Risk factors for pulmonary embolism

Research has demonstrated an increased risk of pulmonary embolism in women associated with obesity, heavy cigarette smoking, and hypertension. Other risk factors include immobilization, surgery within the last 3 months, stroke, history of DVT, and malignancy.

Smoking and ciliary action

Smoking decreases the ciliary action in the tracheobronchial tree, resulting in impaired clearance of respiratory secretions, chronic cough, and frequent respiratory infections.

The nurse instructs a client on the steps needed to obtain a peak expiratory flow rate. In which order should these steps occur? 1. "Take as deep a breath as possible." 2. "Stand up (unless you have a physical disability)." 3. "Place the meter in your mouth, and close your lips around the mouthpiece." 4. "Make sure the device reads zero or is at base level." 5. "Blow out as hard and as fast as possible for 1 to 2 seconds." 6. "Write down the value obtained." 7. "Repeat the process two additional times, and record the highest number in your chart." a. 4, 2, 1, 3, 5, 6, 7 b. 3, 4, 1, 2, 5, 7, 6 c. 2, 1, 3, 4, 5, 6, 7 d. 1, 3, 2, 5, 6, 7, 4

a. 4, 2, 1, 3, 5, 6, 7

Chronic interstitial fibrosis is associated with the development of

adenocarcinoma of the lung

A nurse cares for a client with arthritis who reports frequent asthma attacks. Which action should the nurse take first? a. Review the client's pulmonary function test results. b. Ask about medications the client is currently taking. c. Assess how frequently the client uses a bronchodilator. d. Consult the provider and request arterial blood gases.

b. Ask about medications the client is currently taking.

A nurse administers medications to a client who has asthma. Which medication classification is paired correctly with its physiologic response to the medication? a. Bronchodilator - Stabilizes the membranes of mast cells and prevents the release of inflammatory mediators b. Cholinergic antagonist - Causes bronchodilation by inhibiting the parasympathetic nervous system c. Corticosteroid - Relaxes bronchiolar smooth muscles by binding to and activating pulmonary beta2 receptors d. Cromone - Disrupts the production of pathways of inflammatory mediators

b. Cholinergic antagonist - Causes bronchodilation by inhibiting the parasympathetic nervous system

While assessing a client who is 12 hours postoperative after a thoracotomy for lung cancer, a nurse notices that the lower chest tube is dislodged. Which action should the nurse take first? a. Assess for drainage from the site. b. Cover the insertion site with sterile gauze. c. Contact the provider and obtain a suture kit. d. Reinsert the tube using sterile technique.

b. Cover the insertion site with sterile gauze

The nurse is teaching a client with chronic obstructive pulmonary disease who has been prescribed continuous oxygen therapy at home. Which statement indicates the client correctly understands the teaching? a. "I plan to wear my oxygen when I exercise and feel short of breath." b. "I will use my portable oxygen when grilling burgers in the backyard." c. "I plan to use cotton balls to cushion the oxygen tubing on my ears." d. "I will only smoke while I am wearing my oxygen via nasal cannula."

c. "I plan to use cotton balls to cushion the oxygen tubing on my ears."

A nurse evaluates the following arterial blood gas and vital sign results for a client with chronic obstructive pulmonary disease (COPD): Arterial Blood Gas Results: pH = 7.32 PaCO2 = 62 mm Hg PaO2 = 46 mm Hg HCO3- = 28 mEq/L Vital Signs Heart rate = 110 beats/min Respiratory rate = 12 breaths/min Blood pressure = 145/65 mm Hg Oxygen saturation = 76% Which action should the nurse take first? a. Administer a short-acting beta2 agonist inhaler. b. Document the findings as normal for a client with COPD. c. Teach the client diaphragmatic breathing techniques. d. Initiate oxygenation therapy to increase saturation to 92%.

d. Initiate oxygenation therapy to increase saturation to 92%.

A nurse cares for a client who has a chest tube. When would this client be at highest risk for developing a pneumothorax? a. When the insertion site becomes red and warm to the touch b. When the tube drainage decreases and becomes sanguineous c. When the client experiences pain at the insertion site d. When the tube becomes disconnected from the drainage system

d. When the tube becomes disconnected from the drainage system

A typical physical examination finding for a patient with pneumonia is

increased vocal fremitus on palpation. Other signs of pulmonary consolidation include dullness to percussion, bronchial breath sounds, and crackles in the affected area.

The nurse is caring for a patient admitted to the hospital with pneumonia. Upon assessment, the nurse notes a temperature of 101.4° F, a productive cough with yellow sputum, and a respiratory rate of 20. Which of the following nursing diagnosis is most appropriate based upon this assessment? A) Hyperthermia related to infectious illness B) Ineffective thermoregulation related to chilling C) Ineffective breathing pattern related to pneumonia D) Ineffective airway clearance related to thick secretions

A

The nurse is scheduled to administer seasonal influenza vaccinations to the residents of a long-term care facility. Which of the following criteria would contraindicate the administration of the vaccine to a resident? A) Hypersensitivity to eggs B) Age greater than 80 years C) History of upper respiratory infections D) Chronic obstructive pulmonary disease (COPD)

A

The nurse notices clear nasal drainage in a patient newly admitted with facial trauma, including a nasal fracture. The nurse should A) Test the drainage for the presence of glucose. B) Suction the nose to maintain airway clearance. C) Document the findings and continue monitoring. D) Apply a drip pad and reassure the patient this is normal.

A

When initially teaching a patient the supraglottic swallow following a radical neck dissection, with which of the following foods or fluids should the nurse begin? A) Cola B) Applesauce C) French fries D) White grape juice

A

Which of the following physical assessment findings in a patient with a lower respiratory problem best supports the nursing diagnosis of ineffective airway clearance? A) Basilar crackles B) Respiratory rate of 28 C) Oxygen saturation of 85% D) Presence of greenish sputum

A

The nurse is obtaining a focused respiratory assessment of a 44-year-old female patient who is in severe respiratory distress 2 days after abdominal surgery. What is most important for the nurse to assess? A. Auscultation of bilateral breath sounds B. Percussion of anterior and posterior chest wall C. Palpation of the chest bilaterally for tactile fremitus D. Inspection for anterior and posterior chest expansion

A Rationale: Important assessments obtained during a focused respiratory assessment include auscultation of lung (breath) sounds. Assessment of tactile fremitus has limited value in acute respiratory distress. It is not necessary to assess for both anterior and posterior chest expansion. Percussion of the chest wall is not essential in a focused respiratory assessment.

The patient with Parkinson's disease has a pulse oximetry reading of 72%, but he is not displaying any other signs of decreased oxygenation. What is most likely contributing to his low SpO2 level? A Motion B Anemia C Dark skin color D Thick acrylic nails

A Rationale: Motion is the most likely cause of the low SpO2 for this patient with Parkinson's disease. Anemia, dark skin color, and thick acrylic nails as well as low perfusion, bright fluorescent lights, and intravascular dyes may also cause an inaccurate pulse oximetry result. There is no mention of these or reason to suspect these in this question.

A patient with recurrent shortness of breath has just had a bronchoscopy. What is a priority nursing action immediately following the procedure? A Monitor the patient for laryngeal edema. B Assess the patient's level of consciousness. C Monitor and manage the patient's level of pain. D Assess the patient's heart rate and blood pressure.

A Rationale: Priorities for assessment are the patient's airway and breathing, both of which may be compromised after bronchoscopy by laryngeal edema. These assessment parameters supersede the importance of loss of consciousness (LOC), pain, heart rate, and blood pressure, although the nurse should also be assessing these.

The patient's arterial blood gas results show the PaO2 at 65 mmHg and the SaO2 at 80%. What early manifestations should the nurse expect to observe in this patient? A Restlessness, tachypnea, tachycardia, and diaphoresis B Unexplained confusion, dyspnea at rest, hypotension, and diaphoresis C Combativeness, retractions with breathing, cyanosis, and decreased output D Coma, accessory muscle use, cool and clammy skin, and unexplained fatigue

A Rationale: With inadequate oxygenation, early manifestations include restlessness, tachypnea, tachycardia, and diaphoresis, decreased urinary output, and unexplained fatigue. The unexplained confusion, dyspnea at rest, hypotension, and diaphoresis; combativeness, retractions with breathing, cyanosis, and decreased urinary output; coma, accessory muscle use, cool and clammy skin, and unexplained fatigue occur as later manifestations of inadequate oxygenation.

To promote airway clearance in a patient with pneumonia, the nurse instructs the patient to do which of the following (select all that apply)? A) Maintain adequate fluid intake B) Splint the chest when coughing C) Maintain a high Fowler's position D) Maintain a semi-Fowler's position E) Instruct patient to cough at end of exhalation

A, B, E

A nurse cares for a client who is prescribed an intravenous prostacyclin agent. Which actions should the nurse take to ensure the client's safety while on this medication? (Select all that apply.) a. Keep an intravenous line dedicated strictly to the infusion. b. Teach the client that this medication increases pulmonary pressures. c. Ensure that there is always a backup drug cassette available. d. Start a large-bore peripheral intravenous line. e. Use strict aseptic technique when using the drug delivery system.

A, C, E

During admission of a patient diagnosed with non-small cell carcinoma of the lung, the nurse questions the patient related to a history of which of the following risk factors for this type of cancer (select all that apply)? A) Asbestos exposure B) Cigarette smoking C) Exposure to uranium D) Chronic interstitial fibrosis E) Geographic area in which he was born

A,B,C

When assessing a patient's sleep-rest pattern related to respiratory health, what should the nurse ask the patient about (select all that apply)? A Have trouble falling asleep? B Need to urinate during the night? C Awaken abruptly during the night? D Sleep more than 8 hours per night? E Need to sleep with the head elevated?

A,C,E Rationale: The patient with sleep apnea may have insomnia and/or abrupt awakenings. Patients with cardiovascular disease (e.g., heart failure that may affect respiratory health) may need to sleep with the head elevated on several pillows (orthopnea). Sleeping more than 8 hours per night or needing to urinate during the night is not indicative of impaired respiratory health.

32. A patient who has just been admitted with community-acquired pneumococcal pneumonia has a temperature of 101.6° F with a frequent cough and is complaining of severe pleuritic chest pain. Which prescribed medication should the nurse give first? a. Codeine b. Guaifenesin (Robitussin) c. Acetaminophen (Tylenol) d. Piperacillin/tazobactam (Zosyn)

ANS: D Early initiation of antibiotic therapy has been demonstrated to reduce mortality. The other medications are also appropriate and should be given as soon as possible, but the priority is to start antibiotic therapy.

26. The nurse administers prescribed therapies for a patient with cor pulmonale and right-sided heart failure. Which assessment would best evaluate the effectiveness of the therapies? a. Observe for distended neck veins. b. Auscultate for crackles in the lungs. c. Palpate for heaves or thrills over the heart. d. Review hemoglobin and hematocrit values.

ANS: A Cor pulmonale is right ventricular failure caused by pulmonary hypertension, so clinical manifestations of right ventricular failure such as peripheral edema, jugular venous distention, and right upper-quadrant abdominal tenderness would be expected. Crackles in the lungs are likely to be heard with left-sided heart failure. Findings in cor pulmonale include evidence of right ventricular hypertrophy on electrocardiogram ECG and an increase in intensity of the second heart sound. Heaves or thrills are not common with cor pulmonale. Chronic hypoxemia leads to polycythemia and increased total blood volume and viscosity of the blood. The hemoglobin and hematocrit values are more likely to be elevated with cor pulmonale than decreased.

3. A patient with bacterial pneumonia has rhonchi and thick sputum. What is the nurse's most appropriate action to promote airway clearance? a. Assist the patient to splint the chest when coughing. b. Teach the patient about the need for fluid restrictions. c. Encourage the patient to wear the nasal oxygen cannula. d. Instruct the patient on the pursed lip breathing technique.

ANS: A Coughing is less painful and more likely to be effective when the patient splints the chest during coughing. Fluids should be encouraged to help liquefy secretions. Nasal oxygen will improve gas exchange, but will not improve airway clearance. Pursed lip breathing is used to improve gas exchange in patients with COPD, but will not improve airway clearance.

2. The nurse assesses the chest of a patient with pneumococcal pneumonia. Which finding would the nurse expect? a. Increased tactile fremitus b. Dry, nonproductive cough c. Hyperresonance to percussion d. A grating sound on auscultation

ANS: A Increased tactile fremitus over the area of pulmonary consolidation is expected with bacterial pneumonias. Dullness to percussion would be expected. Pneumococcal pneumonia typically presents with a loose, productive cough. Adventitious breath sounds such as crackles and wheezes are typical. A grating sound is more representative of a pleural friction rub rather than pneumonia.

11. An older patient is receiving standard multidrug therapy for tuberculosis (TB). The nurse should notify the health care provider if the patient exhibits which finding? a. Yellow-tinged skin b. Orange-colored sputum c. Thickening of the fingernails d. Difficulty hearing high-pitched voices

ANS: A Noninfectious hepatitis is a toxic effect of isoniazid (INH), rifampin, and pyrazinamide, and patients who develop hepatotoxicity will need to use other medications. Changes in hearing and nail thickening are not expected with the four medications used for initial TB drug therapy. Presbycusis is an expected finding in the older adult patient. Orange discoloration of body fluids is an expected side effect of rifampin and not an indication to call the health care provider.

31. The nurse cares for a patient who has just had a thoracentesis. Which assessment information obtained by the nurse is a priority to communicate to the health care provider? a. Oxygen saturation is 88%. b. Blood pressure is 145/90 mm Hg. c. Respiratory rate is 22 breaths/minute when lying flat. d. Pain level is 5 (on 0 to 10 scale) with a deep breath.

ANS: A Oxygen saturation would be expected to improve after a thoracentesis. A saturation of 88% indicates that a complication such as pneumothorax may be occurring. The other assessment data also indicate a need for ongoing assessment or intervention, but the low oxygen saturation is the priority.

22. A patient experiences a chest wall contusion as a result of being struck in the chest with a baseball bat. The emergency department nurse would be most concerned if which finding is observed during the initial assessment? a. Paradoxic chest movement b. Complaint of chest wall pain c. Heart rate of 110 beats/minute d. Large bruised area on the chest

ANS: A Paradoxic chest movement indicates that the patient may have flail chest, which can severely compromise gas exchange and can rapidly lead to hypoxemia. Chest wall pain, a slightly elevated pulse rate, and chest bruising all require further assessment or intervention, but the priority concern is poor gas exchange.

42. A patient who was admitted the previous day with pneumonia complains of a sharp pain of 7 (based on 0 to 10 scale) "whenever I take a deep breath." Which action will the nurse take next? a. Auscultate breath sounds. b. Administer the PRN morphine. c. Have the patient cough forcefully. d. Notify the patient's health care provider.

ANS: A The patient's statement indicates that pleurisy or a pleural effusion may have developed and the nurse will need to listen for a pleural friction rub and/or decreased breath sounds. Assessment should occur before administration of pain medications. The patient is unlikely to be able to cough forcefully until pain medication has been administered. The nurse will want to obtain more assessment data before calling the health care provider.

1. Following assessment of a patient with pneumonia, the nurse identifies a nursing diagnosis of ineffective airway clearance. Which assessment data best supports this diagnosis? a. Weak, nonproductive cough effort b. Large amounts of greenish sputum c. Respiratory rate of 28 breaths/minute d. Resting pulse oximetry (SpO2) of 85%

ANS: A The weak, nonproductive cough indicates that the patient is unable to clear the airway effectively. The other data would be used to support diagnoses such as impaired gas exchange and ineffective breathing pattern.

48. The nurse is caring for a patient who has a right-sided chest tube after a right lower lobectomy. Which nursing action can the nurse delegate to the unlicensed assistive personnel (UAP)? a. Document the amount of drainage every eight hours. b. Obtain samples of drainage for culture from the system. c. Assess patient pain level associated with the chest tube. d. Check the water-seal chamber for the correct fluid level.

ANS: A UAP education includes documentation of intake and output. The other actions are within the scope of practice and education of licensed nursing personnel.

2. The nurse is reviewing the medical records for five patients who are scheduled for their yearly physical examinations in September. Which patients should receive the inactivated influenza vaccination (select all that apply)? a. A 76-year-old nursing home resident b. A 36-year-old female patient who is pregnant c. A 42-year-old patient who has a 15 pack-year smoking history d. A 30-year-old patient who takes corticosteroids for rheumatoid arthritis e. A 24-year-old patient who has allergies to penicillin and cephalosporins

ANS: A, B, D Current guidelines suggest that healthy individuals between 6 months and age 49 receive intranasal immunization with live, attenuated influenza vaccine. Individuals who are pregnant, residents of nursing homes, or are immunocompromised or who have chronic medical conditions should receive inactivated vaccine by injection. The corticosteroid use by the 30-year-old increases the risk for infection.

15. When caring for a patient who is hospitalized with active tuberculosis (TB), the nurse observes a student nurse who is assigned to take care of a patient. Which action, if performed by the student nurse, would require an intervention by the nurse? a. The patient is offered a tissue from the box at the bedside. b. A surgical face mask is applied before visiting the patient. c. A snack is brought to the patient from the unit refrigerator. d. Hand washing is performed before entering the patient's room.

ANS: B A high-efficiency particulate-absorbing (HEPA) mask, rather than a standard surgical mask, should be used when entering the patient's room because the HEPA mask can filter out 100% of small airborne particles. Hand washing before entering the patient's room is appropriate. Because anorexia and weight loss are frequent problems in patients with TB, bringing food to the patient is appropriate. The student nurse should perform hand washing after handling a tissue that the patient has used, but no precautions are necessary when giving the patient an unused tissue.

27. A patient with idiopathic pulmonary arterial hypertension (IPAH) is receiving nifedipine (Procardia). Which assessment would best indicate to the nurse that the patient's condition is improving? a. Blood pressure (BP) is less than 140/90 mm Hg. b. Patient reports decreased exertional dyspnea. c. Heart rate is between 60 and 100 beats/minute. d. Patient's chest x-ray indicates clear lung fields.

ANS: B Because a major symptom of IPAH is exertional dyspnea, an improvement in this symptom would indicate that the medication was effective. Nifedipine will affect BP and heart rate, but these parameters would not be used to monitor the effectiveness of therapy for a patient with IPAH. The chest x-ray will show clear lung fields even if the therapy is not effective.

45. An experienced nurse instructs a new nurse about how to care for a patient with dyspnea caused by a pulmonary fungal infection. Which action by the new nurse indicates a need for further teaching? a. Listening to the patient's lung sounds several times during the shift b. Placing the patient on droplet precautions and in a private hospital room c. Increasing the oxygen flow rate to keep the oxygen saturation above 90% d. Monitoring patient serology results to identify the specific infecting organism

ANS: B Fungal infections are not transmitted from person to person. Therefore no isolation procedures are necessary. The other actions by the new nurse are appropriate.

30. A patient has just been admitted with probable bacterial pneumonia and sepsis. Which order should the nurse implement first? a. Chest x-ray via stretcher b. Blood cultures from two sites c. Ciprofloxacin (Cipro) 400 mg IV d. Acetaminophen (Tylenol) rectal suppository

ANS: B Initiating antibiotic therapy rapidly is essential, but it is important that the cultures be obtained before antibiotic administration. The chest x-ray and acetaminophen administration can be done last.

37. The nurse receives change-of-shift report on the following four patients. Which patient should the nurse assess first? a. A 23-year-old patient with cystic fibrosis who has pulmonary function testing scheduled b. A 46-year-old patient on bed rest who is complaining of sudden onset of shortness of breath c. A 77-year-old patient with tuberculosis (TB) who has four antitubercular medications due in 15 minutes d. A 35-year-old patient who was admitted the previous day with pneumonia and has a temperature of 100.2° F (37.8° C)

ANS: B Patients on bed rest who are immobile are at high risk for deep vein thrombosis (DVT). Sudden onset of shortness of breath in a patient with a DVT suggests a pulmonary embolism and requires immediate assessment and action such as oxygen administration. The other patients should also be assessed as soon as possible, but there is no indication that they may need immediate action to prevent clinical deterioration.

4. The nurse provides discharge instructions to a patient who was hospitalized for pneumonia. Which statement, if made by the patient, indicates a good understanding of the instructions? a. "I will call the doctor if I still feel tired after a week." b. "I will continue to do the deep breathing and coughing exercises at home." c. "I will schedule two appointments for the pneumonia and influenza vaccines." d. "I'll cancel my chest x-ray appointment if I'm feeling better in a couple weeks."

ANS: B Patients should continue to cough and deep breathe after discharge. Fatigue is expected for several weeks. The Pneumovax and influenza vaccines can be given at the same time in different arms. Explain that a follow-up chest x-ray needs to be done in 6 to 8 weeks to evaluate resolution of pneumonia.

9. The nurse teaches a patient about the transmission of pulmonary tuberculosis (TB). Which statement, if made by the patient, indicates that teaching was effective? a. "I will avoid being outdoors whenever possible." b. "My husband will be sleeping in the guest bedroom." c. "I will take the bus instead of driving to visit my friends." d. "I will keep the windows closed at home to contain the germs."

ANS: B Teach the patient how to minimize exposure to close contacts and household members. Homes should be well ventilated, especially the areas where the infected person spends a lot of time. While still infectious, the patient should sleep alone, spend as much time as possible outdoors, and minimize time in congregate settings or on public transportation.

16. The nurse obtains the following assessment data on an older patient who has influenza. Which information will be most important for the nurse to communicate to the health care provider? a. Fever of 100.4° F (38° C) b. Diffuse crackles in the lungs c. Sore throat and frequent cough d. Myalgia and persistent headache

ANS: B The crackles indicate that the patient may be developing pneumonia, a common complication of influenza, which would require aggressive treatment. Myalgia, headache, mild temperature elevation, and sore throat with cough are typical manifestations of influenza and are treated with supportive care measures such as over-the-counter (OTC) pain relievers and increased fluid intake.

39. A patient is admitted to the emergency department with an open stab wound to the left chest. What is the first action that the nurse should take? a. Position the patient so that the left chest is dependent. b. Tape a nonporous dressing on three sides over the chest wound. c. Cover the sucking chest wound firmly with an occlusive dressing. d. Keep the head of the patient's bed at no more than 30 degrees elevation.

ANS: B The dressing taped on three sides will allow air to escape when intrapleural pressure increases during expiration, but it will prevent air from moving into the pleural space during inspiration. Placing the patient on the left side or covering the chest wound with an occlusive dressing will allow trapped air in the pleural space and cause tension pneumothorax. The head of the bed should be elevated to 30 to 45 degrees to facilitate breathing.

13. After 2 months of tuberculosis (TB) treatment with isoniazid (INH), rifampin (Rifadin), pyrazinamide (PZA), and ethambutol, a patient continues to have positive sputum smears for acid-fast bacilli (AFB). Which action should the nurse take next? a. Teach about treatment for drug-resistant TB treatment. b. Ask the patient whether medications have been taken as directed. c. Schedule the patient for directly observed therapy three times weekly. d. Discuss with the health care provider the need for the patient to use an injectable antibiotic.

ANS: B The first action should be to determine whether the patient has been compliant with drug therapy because negative sputum smears would be expected if the TB bacillus is susceptible to the medications and if the medications have been taken correctly. Assessment is the first step in the nursing process. Depending on whether the patient has been compliant or not, different medications or directly observed therapy may be indicated. The other options are interventions based on assumptions until an assessment has been completed.

21. The nurse monitors a patient after chest tube placement for a hemopneumothorax. The nurse is most concerned if which assessment finding is observed? a. A large air leak in the water-seal chamber b. 400 mL of blood in the collection chamber c. Complaint of pain with each deep inspiration d. Subcutaneous emphysema at the insertion site

ANS: B The large amount of blood may indicate that the patient is in danger of developing hypovolemic shock. An air leak would be expected immediately after chest tube placement for a pneumothorax. Initially, brisk bubbling of air occurs in this chamber when a pneumothorax is evacuated. The pain should be treated but is not as urgent a concern as the possibility of continued hemorrhage. Subcutaneous emphysema should be monitored but is not unusual in a patient with pneumothorax. A small amount of subcutaneous air is harmless and will be reabsorbed.

A nurse assesses a client with asthma and notes bilateral wheezing, decreased pulse oxygen saturation, and suprasternal retraction on inhalation. Which actions should the nurse take? (Select all that apply.) a. Administer prescribed salmeterol (Serevent) inhaler. b. Assess the client for a tracheal deviation. c. Administer oxygen to keep saturations greater than 94%. d. Perform peak expiratory flow readings. e. Administer prescribed albuterol (Proventil) inhaler.

C, E

5. The nurse develops a plan of care to prevent aspiration in a high-risk patient. Which nursing action will be most effective? a. Turn and reposition immobile patients at least every 2 hours. b. Place patients with altered consciousness in side-lying positions. c. Monitor for respiratory symptoms in patients who are immunosuppressed. d. Insert nasogastric tube for feedings for patients with swallowing problems.

ANS: B The risk for aspiration is decreased when patients with a decreased level of consciousness are placed in a side-lying or upright position. Frequent turning prevents pooling of secretions in immobilized patients but will not decrease the risk for aspiration in patients at risk. Monitoring of parameters such as breath sounds and oxygen saturation will help detect pneumonia in immunocompromised patients, but it will not decrease the risk for aspiration. Conditions that increase the risk of aspiration include decreased level of consciousness (e.g., seizure, anesthesia, head injury, stroke, alcohol intake), difficulty swallowing, and nasogastric intubation with or without tube feeding. With loss of consciousness, the gag and cough reflexes are depressed, and aspiration is more likely to occur. Other high-risk groups are those who are seriously ill, have poor dentition, or are receiving acid-reducing medications.

28. A patient with a pleural effusion is scheduled for a thoracentesis. Which action should the nurse take to prepare the patient for the procedure? a. Start a peripheral IV line to administer the necessary sedative drugs. b. Position the patient sitting upright on the edge of the bed and leaning forward. c. Obtain a large collection device to hold 2 to 3 liters of pleural fluid at one time. d. Remove the water pitcher and remind the patient not to eat or drink anything for 6 hours.

ANS: B When the patient is sitting up, fluid accumulates in the pleural space at the lung bases and can more easily be located and removed. The patient does not usually require sedation for the procedure, and there are no restrictions on oral intake because the patient is not sedated or unconscious. Usually only 1000 to 1200 mL of pleural fluid is removed at one time. Rapid removal of a large volume can result in hypotension, hypoxemia, or pulmonary edema.

24. A patient who has a right-sided chest tube following a thoracotomy has continuous bubbling in the suction-control chamber of the collection device. Which action by the nurse is most appropriate? a. Document the presence of a large air leak. b. Notify the surgeon of a possible pneumothorax. c. Take no further action with the collection device. d. Adjust the dial on the wall regulator to decrease suction.

ANS: C Continuous bubbling is expected in the suction-control chamber and indicates that the suction-control chamber is connected to suction. An air leak would be detected in the water-seal chamber. There is no evidence of pneumothorax. Increasing or decreasing the vacuum source will not adjust the suction pressure. The amount of suction applied is regulated by the amount of water in this chamber and not by the amount of suction applied to the system.

43. A patient has acute bronchitis with a nonproductive cough and wheezes. Which topic should the nurse plan to include in the teaching plan? a. Purpose of antibiotic therapy b. Ways to limit oral fluid intake c. Appropriate use of cough suppressants d. Safety concerns with home oxygen therapy

ANS: C Cough suppressants are frequently prescribed for acute bronchitis. Because most acute bronchitis is viral in origin, antibiotics are not prescribed unless there are systemic symptoms. Fluid intake is encouraged. Home oxygen is not prescribed for acute bronchitis, although it may be used for chronic bronchitis.

33. A patient is diagnosed with both human immunodeficiency virus (HIV) and active tuberculosis (TB) disease. Which information obtained by the nurse is most important to communicate to the health care provider? a. The Mantoux test had an induration of 7 mm. b. The chest-x-ray showed infiltrates in the lower lobes. c. The patient is being treated with antiretrovirals for HIV infection. d. The patient has a cough that is productive of blood-tinged mucus.

ANS: C Drug interactions can occur between the antiretrovirals used to treat HIV infection and the medications used to treat TB. The other data are expected in a patient with HIV and TB.

25. The nurse provides preoperative instruction for a patient scheduled for a left pneumonectomy for cancer of the lung. Which information should the nurse include about the patient's postoperative care? a. Positioning on the right side b. Bed rest for the first 24 hours c. Frequent use of an incentive spirometer d. Chest tube placement with continuous drainage

ANS: C Frequent deep breathing and coughing are needed after chest surgery to prevent atelectasis. To promote gas exchange, patients after pneumonectomy are positioned on the surgical side. Early mobilization decreases the risk for postoperative complications such as pneumonia and deep vein thrombosis. In a pneumonectomy, chest tubes may or may not be placed in the space from which the lung was removed. If a chest tube is used, it is clamped and only released by the surgeon to adjust the volume of serosanguineous fluid that will fill the space vacated by the lung. If the cavity overfills, it could compress the remaining lung and compromise the cardiovascular and pulmonary function. Daily chest x-rays can be used to assess the volume and space.

10. A patient who is taking rifampin (Rifadin) for tuberculosis calls the clinic and reports having orange discolored urine and tears. Which is the best response by the nurse? a. Ask if the patient is experiencing shortness of breath, hives, or itching. b. Ask the patient about any visual abnormalities such as red-green color discrimination. c. Explain that orange discolored urine and tears are normal while taking this medication. d. Advise the patient to stop the drug and report the symptoms to the health care provider.

ANS: C Orange-colored body secretions are a side effect of rifampin. The patient does not have to stop taking the medication. The findings are not indicative of an allergic reaction. Alterations in red-green color discrimination commonly occurs when taking ethambutol (Myambutol), which is a different TB medication.

7. The health care provider writes an order for bacteriologic testing for a patient who has a positive tuberculosis skin test. Which action should the nurse take? a. Teach about the reason for the blood tests. b. Schedule an appointment for a chest x-ray. c. Teach about the need to get sputum specimens for 2 to 3 consecutive days. d. Instruct the patient to expectorate three specimens as soon as possible.

ANS: C Sputum specimens are obtained on 2 to 3 consecutive days for bacteriologic testing for M. tuberculosis. The patient should not provide all the specimens at once. Blood cultures are not used for tuberculosis testing. A chest x-ray is not bacteriologic testing. Although the findings on chest x-ray examination are important, it is not possible to make a diagnosis of TB solely based on chest x-ray findings because other diseases can mimic the appearance of TB.

41. The nurse is caring for a patient with idiopathic pulmonary arterial hypertension (IPAH) who is receiving epoprostenol (Flolan). Which assessment information requires the most immediate action by the nurse? a. The oxygen saturation is 94%. b. The blood pressure is 98/56 mm Hg. c. The patient's central IV line is disconnected. d. The international normalized ratio (INR) is prolonged.

ANS: C The half-life of this drug is 6 minutes, so the nurse will need to restart the infusion as soon as possible to prevent rapid clinical deterioration. The other data also indicate a need for ongoing monitoring or intervention, but the priority action is to reconnect the infusion.

44. Which action by the nurse will be most effective in decreasing the spread of pertussis in a community setting? a. Providing supportive care to patients diagnosed with pertussis b. Teaching family members about the need for careful hand washing c. Teaching patients about the need for adult pertussis immunizations d. Encouraging patients to complete the prescribed course of antibiotics

ANS: C The increased rate of pertussis in adults is thought to be due to decreasing immunity after childhood immunization. Immunization is the most effective method of protecting communities from infectious diseases. Hand washing should be taught, but pertussis is spread by droplets and contact with secretions. Supportive care does not shorten the course of the disease or the risk for transmission. Taking antibiotics as prescribed does assist with decreased transmission, but patients are likely to have already transmitted the disease by the time the diagnosis is made.

6. A patient with right lower-lobe pneumonia has been treated with IV antibiotics for 3 days. Which assessment data obtained by the nurse indicates that the treatment has been effective? a. Bronchial breath sounds are heard at the right base. b. The patient coughs up small amounts of green mucus. c. The patient's white blood cell (WBC) count is 9000/µL. d. Increased tactile fremitus is palpable over the right chest.

ANS: C The normal WBC count indicates that the antibiotics have been effective. All the other data suggest that a change in treatment is needed.

Which physical assessment finding in a patient with a lower respiratory problem best supports the nursing diagnosis of ineffective airway clearance? A. Basilar crackles B. Respiratory rate of 28 C. Oxygen saturation of 85% D. Presence of greenish sputum

Ans: A The presence of adventitious breath sounds indicates that there is accumulation of secretions in the lower airways. This would be consistent with a nursing diagnosis of ineffective airway clearance because the patient is retaining secretions. The rapid respiratory rate, low oxygen saturation, and presence of greenish sputum may occur with a lower respiratory problem, but do not definitely support the nursing diagnosis of ineffective airway clearance.

14. Employee health test results reveal a tuberculosis (TB) skin test of 16-mm induration and a negative chest x-ray for a staff nurse working on the pulmonary unit. The nurse has no symptoms of TB. Which information should the occupational health nurse plan to teach the staff nurse? a. Standard four-drug therapy for TB b. Need for annual repeat TB skin testing c. Use and side effects of isoniazid (INH) d. Bacille Calmette-Guérin (BCG) vaccine

ANS: C The nurse is considered to have a latent TB infection and should be treated with INH daily for 6 to 9 months. The four-drug therapy would be appropriate if the nurse had active TB. TB skin testing is not done for individuals who have already had a positive skin test. BCG vaccine is not used in the United States for TB and would not be helpful for this individual, who already has a TB infection.

4. A nurse who is caring for patient with a tracheostomy tube in place has just auscultated rhonchi bilaterally. If the patient is unsuccessful in coughing up secretions, what action should the nurse take? a. Encourage increased incentive spirometer use. b. Encourage the patient to increase oral fluid intake. c. Put on sterile gloves and use a sterile catheter to suction. d. Preoxygenate the patient for 3 minutes before suctioning.

ANS: C This patient needs suctioning now to secure a patent airway. Sterile gloves and a sterile catheter are used when suctioning a tracheostomy. Preoxygenation for 3 minutes is not necessary. Incentive spirometer (IS) use opens alveoli and can induce coughing, which can mobilize secretions. However, the patient with a tracheostomy may not be able to use an incentive spirometer. Increasing oral fluid intake would not moisten and help mobilize secretions in a timely manner.

34. A patient with pneumonia has a fever of 101.4° F (38.6° C), a nonproductive cough, and an oxygen saturation of 88%. The patient complains of weakness, fatigue, and needs assistance to get out of bed. Which nursing diagnosis should the nurse assign as the highest priority? a. Hyperthermia related to infectious illness b. Impaired transfer ability related to weakness c. Ineffective airway clearance related to thick secretions d. Impaired gas exchange related to respiratory congestion

ANS: D All these nursing diagnoses are appropriate for the patient, but the patient's oxygen saturation indicates that all body tissues are at risk for hypoxia unless the gas exchange is improved.

12. An alcoholic and homeless patient is diagnosed with active tuberculosis (TB). Which intervention by the nurse will be most effective in ensuring adherence with the treatment regimen? a. Arrange for a friend to administer the medication on schedule. b. Give the patient written instructions about how to take the medications. c. Teach the patient about the high risk for infecting others unless treatment is followed. d. Arrange for a daily noon meal at a community center where the drug will be administered.

ANS: D Directly observed therapy is the most effective means for ensuring compliance with the treatment regimen, and arranging a daily meal will help ensure that the patient is available to receive the medication. The other nursing interventions may be appropriate for some patients but are not likely to be as helpful for this patient.

46. Which intervention will the nurse include in the plan of care for a patient who is diagnosed with a lung abscess? a. Teach the patient to avoid the use of over-the-counter expectorants. b. Assist the patient with chest physiotherapy and postural drainage. c. Notify the health care provider immediately about any bloody or foul-smelling sputum. d. Teach about the need for prolonged antibiotic therapy after discharge from the hospital.

ANS: D Long-term antibiotic therapy is needed for effective eradication of the infecting organisms in lung abscess. Chest physiotherapy and postural drainage are not recommended for lung abscess because they may lead to spread of the infection. Foul smelling and bloody sputum are common clinical manifestations in lung abscess. Expectorants may be used because the patient is encouraged to cough.

8. A patient is admitted with active tuberculosis (TB). The nurse should question a health care provider's order to discontinue airborne precautions unless which assessment finding is documented? a. Chest x-ray shows no upper lobe infiltrates. b. TB medications have been taken for 6 months. c. Mantoux testing shows an induration of 10 mm. d. Three sputum smears for acid-fast bacilli are negative.

ANS: D Negative sputum smears indicate that Mycobacterium tuberculosis is not present in the sputum, and the patient cannot transmit the bacteria by the airborne route. Chest x-rays are not used to determine whether treatment has been successful. Taking medications for 6 months is necessary, but the multidrug-resistant forms of the disease might not be eradicated after 6 months of therapy. Repeat Mantoux testing would not be done because the result will not change even with effective treatment.

38. The nurse is performing tuberculosis (TB) skin tests in a clinic that has many patients who have immigrated to the United States. Which question is most important for the nurse to ask before the skin test? a. "Is there any family history of TB?" b. "How long have you lived in the United States?" c. "Do you take any over-the-counter (OTC) medications?" d. "Have you received the bacille Calmette-Guérin (BCG) vaccine for TB?"

ANS: D Patients who have received the BCG vaccine will have a positive Mantoux test. Another method for screening (such as a chest x-ray) will need to be used in determining whether the patient has a TB infection. The other information also may be valuable but is not as pertinent to the decision about doing TB skin testing.

35. The nurse supervises unlicensed assistive personnel (UAP) who are providing care for a patient with right lower lobe pneumonia. The nurse should intervene if which action by UAP is observed? a. UAP splint the patient's chest during coughing. b. UAP assist the patient to ambulate to the bathroom. c. UAP help the patient to a bedside chair for meals. d. UAP lower the head of the patient's bed to 15 degrees.

ANS: D Positioning the patient with the head of the bed lowered will decrease ventilation. The other actions are appropriate for a patient with pneumonia.

47. The nurse provides discharge teaching for a patient who has two fractured ribs from an automobile accident. Which statement, if made by the patient, would indicate that teaching has been effective? a. "I am going to buy a rib binder to wear during the day." b. "I can take shallow breaths to prevent my chest from hurting." c. "I should plan on taking the pain pills only at bedtime so I can sleep." d. "I will use the incentive spirometer every hour or two during the day."

ANS: D Prevention of the complications of atelectasis and pneumonia is a priority after rib fracture. This can be ensured by deep breathing and coughing. Use of a rib binder, shallow breathing, and taking pain medications only at night are likely to result in atelectasis.

36. A patient with a possible pulmonary embolism complains of chest pain and difficulty breathing. The nurse finds a heart rate of 142 beats/minute, blood pressure of 100/60 mmHg, and respirations of 42 breaths/minute. Which action should the nurse take first? a. Administer anticoagulant drug therapy. b. Notify the patient's health care provider. c. Prepare patient for a spiral computed tomography (CT). d. Elevate the head of the bed to a semi-Fowler's position.

ANS: D The patient has symptoms consistent with a pulmonary embolism (PE). Elevating the head of the bed will improve ventilation and gas exchange. The other actions can be accomplished after the head is elevated (and oxygen is started). A spiral CT may be ordered by the health care provider to identify PE. Anticoagulants may be ordered after confirmation of the diagnosis of PE.

19. An hour after a thoracotomy, a patient complains of incisional pain at a level 7 (based on 0 to 10 scale) and has decreased left-sided breath sounds. The pleural drainage system has 100 mL of bloody drainage and a large air leak. Which action is best for the nurse to take next? a. Milk the chest tube gently to remove any clots. b. Clamp the chest tube momentarily to check for the origin of the air leak. c. Assist the patient to deep breathe, cough, and use the incentive spirometer. d. Set up the patient controlled analgesia (PCA) and administer the loading dose of morphine.

ANS: D The patient is unlikely to take deep breaths or cough until the pain level is lower. A chest tube output of 100 mL is not unusual in the first hour after thoracotomy and would not require milking of the chest tube. An air leak is expected in the initial postoperative period after thoracotomy.

23. When assessing a patient who has just arrived after an automobile accident, the emergency department nurse notes tachycardia and absent breath sounds over the right lung. For which intervention will the nurse prepare the patient? a. Emergency pericardiocentesis b. Stabilization of the chest wall with tape c. Administration of an inhaled bronchodilator d. Insertion of a chest tube with a chest drainage system

ANS: D The patient's history and absent breath sounds suggest a right-sided pneumothorax or hemothorax, which will require treatment with a chest tube and drainage. The other therapies would be appropriate for an acute asthma attack, flail chest, or cardiac tamponade, but the patient's clinical manifestations are not consistent with these problems.

49. After change-of-shift report, which patient should the nurse assess first? a. 72-year-old with cor pulmonale who has 4+ bilateral edema in his legs and feet b. 28-year-old with a history of a lung transplant and a temperature of 101° F (38.3° C) c. 40-year-old with a pleural effusion who is complaining of severe stabbing chest pain d. 64-year-old with lung cancer and tracheal deviation after subclavian catheter insertion

ANS: D The patient's history and symptoms suggest possible tension pneumothorax, a medical emergency. The other patients also require assessment as soon as possible, but tension pneumothorax will require immediate treatment to avoid death from inadequate cardiac output or hypoxemia.

The school nurse is providing information to high school students about influenza prevention. What should the nurse emphasize in teaching to prevent the transmission of the virus (select all that apply)? A. Cover the nose when coughing. B. Obtain an influenza vaccination. C. Stay at home when symptomatic. D. Drink non-caffeinated fluids daily. E. Obtain antibiotic therapy promptly.

Ans: A, B, C Covering the nose and mouth when coughing is an effective way to prevent the spread of the virus. Obtaining an influenza vaccination helps prevent the flu. Staying at home helps prevent direct exposure of others to the virus. Drinking fluids helps liquefy secretions but does not prevent influenza. Antibiotic therapy is not used unless the patient develops a secondary bacterial infection.

The nurse is scheduled to administer seasonal influenza vaccinations to the residents of a long-term care facility. What would be a contraindication to the administration of the vaccine to a resident? A. Hypersensitivity to eggs B. Age greater than 80 years C. History of upper respiratory infections D. Chronic obstructive pulmonary disease (COPD)

Ans: A Although current vaccines are highly purified, and reactions are extremely uncommon, a hypersensitivity to eggs precludes vaccination because the vaccine is produced in eggs. Advanced age and a history of respiratory illness are not contraindications for influenza vaccination.

The nurse is caring for a patient admitted to the hospital with pneumonia. Upon assessment, the nurse notes a temperature of 101.4° F, a productive cough with yellow sputum, and a respiratory rate of 20. Which nursing diagnosis is most appropriate based upon this assessment? A. Hyperthermia related to infectious illness B. Ineffective thermoregulation related to chilling C. Ineffective breathing pattern related to pneumonia D. Ineffective airway clearance related to thick secretions

Ans: A Because the patient has spiked a temperature and has a diagnosis of pneumonia, the logical nursing diagnosis is hyperthermia related to infectious illness. There is no evidence of a chill, and her breathing pattern is within normal limits at 20 breaths/minute. There is no evidence of ineffective airway clearance from the information given because the patient is expectorating sputum.

The nurse notices clear nasal drainage in a patient newly admitted with facial trauma, including a nasal fracture. What should the nurse do first? A. Test the drainage for the presence of glucose. B. Suction the nose to maintain airway clearance. C. Document the findings and continue monitoring. D. Apply a drip pad and reassure the patient this is normal.

Ans: A Clear nasal drainage suggests leakage of cerebrospinal fluid (CSF). The drainage should be tested for the presence of glucose, which would indicate the presence of CSF. Suctioning should not be done. Documenting the findings and monitoring are important after notifying the health care provider. A drip pad may be applied, but the patient should not be reassured that this is normal.

A 73-year-old female patient who lives alone is admitted to the hospital with a diagnosis of pneumococcal pneumonia. Which clinical manifestation, if observed by the nurse, indicates that the patient is likely to be hypoxic? A. Sudden onset of confusion B. Oral temperature of 102.3o F C. Coarse crackles in lung bases D. Clutching chest on inspiration

Ans: A Confusion or stupor (related to hypoxia) may be the only clinical manifestation of pneumonia in an older adult patient. An elevated temperature, coarse crackles, and pleuritic chest pain with guarding may occur with pneumonia, but these symptoms do not indicate hypoxia.

A 24-year-old male with a gunshot wound to the right side of the chest walks into the emergency department while leaning on another young man. The patient exhibits severe shortness of breath and decreased breath sounds on the right side. Which action should the nurse take immediately? A. Cover the chest wound with a nonporous dressing taped on three sides. B. Pack the chest wound with sterile saline soaked gauze and tape securely. C. Stabilize the chest wall with tape and initiate positive pressure ventilation. D. Apply a pressure dressing over the wound to prevent excessive loss of blood.

Ans: A The patient has a sucking chest wound (open pneumothorax). Air enters the pleural space through the chest wall during inspiration. Emergency treatment consists of covering the wound with an occlusive dressing that is secured on three sides. During inspiration the dressing pulls against the wound preventing air from entering the pleural space. During expiration the dressing is pushed out and air escapes through the wound and from under the dressing.

To promote airway clearance in a patient with pneumonia, what should the nurse instruct the patient to do (select all that apply)? A. Maintain adequate fluid intake. B. Splint the chest when coughing. C. Maintain a 30-degree elevation. D. Maintain a semi-Fowler's position. E. Instruct patient to cough at end of exhalation.*

Ans: A, B, E Maintaining adequate fluid intake liquefies secretions, allowing easier expectoration. The nurse should instruct the patient to splint the chest while coughing. This will reduce discomfort and allow for a more effective cough. Coughing at the end of exhalation promotes a more effective cough. The patient should be positioned in an upright sitting position (high Fowler's) with head slightly flexed.

A patient whose tracheostomy was inserted 30 minutes ago is recovering in the postanesthesia recovery unit when he coughs and expels the tracheostomy tube. How should the nurse respond? A. Suction the tracheostomy opening. B. Maintain the airway with a sterile hemostat. C. Use an Ambu bag and mask to ventilate the patient. D. Insert the tracheostomy tube obturator into the stoma.

Ans: B As long as the patient is not in acute respiratory distress after dislodging the tracheostomy tube, the nurse should use a sterile hemostat to maintain an open airway until a sterile tracheostomy tube can be reinserted into the tracheal opening. The tracheostomy is an open surgical wound that has not had time to mature into a stoma. If the patient is in respiratory distress, the nurse will use an Ambu bag and mask to ventilate the patient temporarily.

A 62-year-old male has a tracheostomy tube after reconstructive surgery for invasive head and neck cancer. What is most important for the nurse to assess before performing tracheostomy cannula care? A. Level of consciousness B. Quality of breath sounds C. Presence of the gag reflex D. Tracheostomy cuff pressure

Ans: B Before performing tracheostomy care, the nurse will auscultate lung sounds to determine the presence of secretions. To prevent aspiration, secretions must be cleared either by coughing or by suctioning before performing tracheostomy cannula care.

Which task can the registered nurse (RN) delegate to unlicensed assistive personnel (UAP) in the care of a stable patient who has a tracheostomy? A. Assessing the need for suctioning B. Suctioning the patient's oropharynx C. Assessing the patient's swallowing ability D. Maintaining appropriate cuff inflation pressure

Ans: B Providing the individual has been trained in correct technique, UAP may suction the patient's oropharynx. Assessing the need for suctioning should be performed by an RN or licensed practical nurse, whereas swallowing assessment and the maintenance of cuff inflation pressure should be performed solely by the RN.

When caring for a patient who is 3 hours postoperative laryngectomy, what is the nurse's highestpriority assessment? A. Patient comfort B. Airway patency C. Incisional drainage D. Blood pressure and heart rate

Ans: B Remember the ABCs with prioritization. Airway patency is always the highest priority and is essential for a patient undergoing surgery surrounding the upper respiratory system. Comfort, drainage, and vital signs follow the ABCs in priority.

The nurse in the occupational health clinic prepares to administer the influenza vaccine by nasal spray to a 35-year-old female employee. Which question should the nurse ask before administration of this vaccine? A. "Are you allergic to chicken?" B. "Could you be pregnant now?" C. "Did you ever have influenza?" D. "Have you ever had hepatitis B?"

Ans: B The live attenuated influenza vaccine (LAIV) is given by nasal spray and approved for healthy people age 2 years to 49 years. The LAIV is given only to nonpregnant, healthy people. The inactivated vaccine is given by injection and is approved for use in people 6 months or older. The inactivated vaccine can be used in pregnancy, in people with chronic conditions, or in people who are immunosuppressed. Influenza vaccination is contraindicated if the person has a history of Guillain-Barré syndrome or a hypersensitivity to eggs.

After admitting a patient from home to the medical unit with a diagnosis of pneumonia, which physician orders will the nurse verify have been completed before administering a dose of cefuroxime (Ceftin) to the patient? A. Orthostatic blood pressures B. Sputum culture and sensitivity C. Pulmonary function evaluation D. Serum laboratory studies ordered for AM

Ans: B The nurse should ensure that the sputum for culture and sensitivity was sent to the laboratory before administering the cefuroxime as this is community-acquired pneumonia. It is important that the organisms are correctly identified (by the culture) before the antibiotic takes effect. The test will also determine whether the proper antibiotic has been ordered (sensitivity testing). Although antibiotic administration should not be unduly delayed while waiting for the patient to expectorate sputum, orthostatic BP, pulmonary function evaluation, and serum laboratory tests will not be affected by the administration of antibiotics.

The nurse cares for a 50-year-old patient with pneumonia that has been unresponsive to two different antibiotics. Which task is most important for the nurse to complete before administering a newly prescribed antibiotic? A. Teach the patient to cough and deep breathe. B. Take the temperature, pulse, and respiratory rate. C. Obtain a sputum specimen for culture and Gram stain. D. Check the patient's oxygen saturation by pulse oximetry.

Ans: C A sputum specimen for culture and Gram stain to identify the organism should be obtained before beginning antibiotic therapy. However, antibiotic administration should not be delayed if a specimen cannot be readily obtained because delays in antibiotic therapy can increase morbidity and mortality risks.

Which clinical manifestation should the nurse expect to find during assessment of a patient admitted with pneumonia? A. Hyperresonance on percussion B. Vesicular breath sounds in all lobes C. Increased vocal fremitus on palpation D. Fine crackles in all lobes on auscultation

Ans: C A typical physical examination finding for a patient with pneumonia is increased vocal fremitus on palpation. Other signs of pulmonary consolidation include bronchial breath sounds, egophony, and crackles in the affected area. With pleural effusion, there may be dullness to percussion over the affected area.

Which patient is at highest risk of aspiration? A. A 58-year-old patient with absent bowel sounds 12 hours after abdominal surgery B. A 67-year-old patient who had a cerebrovascular accident with expressive dysphasia C. A 26-year-old patient with continuous enteral tube feedings through a nasogastric tube D. A 52-year-old patient with viral pneumonia and coarse crackles throughout the lung fields

Ans: C Conditions that increase the risk of aspiration include decreased level of consciousness, difficulty swallowing (dysphagia), and nasogastric intubation with or without tube feeding. With loss of consciousness, the gag and cough reflexes are depressed, and aspiration is more likely to occur. Dysphasia is difficulty with speech. Absent bowel sounds and coarse crackles do not increase the risk for aspiration.

What is the priority nursing assessment in the care of a patient who has a tracheostomy? A. Electrolyte levels and daily weights B. Assessment of speech and swallowing C. Respiratory rate and oxygen saturation D. Pain assessment and assessment of mobility

Ans: C The priority assessment in the care of a patient with a tracheostomy focuses on airway and breathing. These assessments supersede the nurse's assessments that may also be necessary, such as nutritional status, speech, pain, and swallowing ability.

What nursing intervention is most appropriate to enhance oxygenation in a patient with unilateral malignant lung disease? A. Positioning patient on right side B. Maintaining adequate fluid intake C. Positioning patient with "good lung" down D. Performing postural drainage every 4 hours

Ans: C Therapeutic positioning identifies the best position for the patient, thus assuring stable oxygenation status. Research indicates that positioning the patient with the unaffected lung (good lung) dependent best promotes oxygenation in patients with unilateral lung disease. For bilateral lung disease, the right lung down has best ventilation and perfusion. Increasing fluid intake and performing postural drainage will facilitate airway clearance, but positioning is most appropriate to enhance oxygenation.

What is the priority nursing intervention in helping a patient expectorate thick lung secretions? A. Humidify the oxygen as able. B. Administer cough suppressant q4hr. C. Teach patient to splint the affected area. D. Increase fluid intake to 3 L/day if tolerated.

Ans: D Although several interventions may help the patient expectorate mucus, the highest priority should be on increasing fluid intake, which will liquefy the secretions so that the patient can expectorate them more easily. Humidifying the oxygen is also helpful but is not the primary intervention. Teaching the patient to splint the affected area may also be helpful in decreasing discomfort but does not assist in expectoration of thick secretions.

A 71-year-old patient is admitted with acute respiratory distress related to cor pulmonale. Which nursing intervention is most appropriate during admission of this patient? A. Perform a comprehensive health history with the patient to review prior respiratory problems. B. Complete a full physical examination to determine the effect of the respiratory distress on other body functions. C. Delay any physical assessment of the patient and review with the family the patient's history of respiratory problems. D. Perform a physical assessment of the respiratory system and ask specific questions related to this episode of respiratory distress

Ans: D Because the patient is having respiratory difficulty, the nurse should ask specific questions about this episode and perform a physical assessment of this system. Further history taking and physical examination of other body systems can proceed once the patient's acute respiratory distress is being managed.

The nurse teaches a 20-year-old female patient who is prescribed budesonide (Rhinocort) intranasal spray for seasonal allergic rhinitis. The nurse determines that medication teaching is successful if the patient makes which statement? A. "My liver function will be checked with blood tests every 2 to 3 months." B. "The medication will decrease the congestion within 3 to 5 minutes after use." C. "I may develop a serious infection because the medication reduces my immunity." D. "I will use the medication every day of the season whether I have symptoms or not."

Ans: D Budesonide should be started 2 weeks before pollen season starts and used on a regular basis, and not as needed. The spray acts to decrease inflammation and the effect is not immediate as with decongestant sprays. At recommended doses, budesonide has only local effects and will not result in immunosuppression or a systemic infection. Zafirlukast (Accolate) is a leukotriene receptor antagonist and may alter liver function tests (LFTs). LFTs must be monitored periodically in the patient taking zafirlukast.

All of the following care tasks are needed by a patient admitted for joint replacement surgery who has had a permanent tracheostomy for over 10 years. Which task is appropriate for the nurse to delegate to unlicensed assistive personnel (UAP)? A. Suction the tracheostomy. B. Check stoma site for skin breakdown. C. Complete tracheostomy care using sterile technique. D. Provide oral care with a toothbrush and tonsil suction tube.

Ans: D Oral care (for a stable patient with a tracheostomy) can be delegated to UAP. A registered nurse would be responsible for assessments (e.g., checking the stoma for skin breakdown) and tracheostomy suctioning and care.

When planning appropriate nursing interventions for a patient with metastatic lung cancer and a 60-pack-per-year history of cigarette smoking, the nurse recognizes that the smoking has most likely decreased the patient's underlying respiratory defenses because of impairment of which of the following? A) Cough reflex B) Mucociliary clearance C) Reflex bronchoconstriction D) Ability to filter particles from the air

B

Which of the following tasks can the registered nurse (RN) delegate to nursing assistive personnel (NAP) in the care of a stable patient who has a tracheostomy? A) Assessing the need for suctioning B) Suctioning the patient's oropharynx C) Assessing the patient's swallowing ability D) Maintaining appropriate cuff inflation pressure

B

After assisting at the bedside with a thoracentesis, the nurse should continue to assess the patient for signs and symptoms of what? A Bronchospasm B Pneumothorax C Pulmonary edema D Respiratory acidosis

B Rationale: Because thoracentesis involves the introduction of a catheter into the pleural space, there is a risk of pneumothorax. Thoracentesis does not carry a significant potential for causing bronchospasm, pulmonary edema, or respiratory acidosis.

Which patient is exhibiting an early clinical manifestation of hypoxemia? A. A 48-year-old patient who is intoxicated and acutely disoriented to time and place B. A 72-year-old patient who has four new premature ventricular contractions per minute C. A 67-year-old patient who has dyspnea while resting in the bed or in a reclining chair D. A 94-year-old patient who has renal insufficiency, anemia, and decreased urine output

B Rationale: Early clinical manifestations of hypoxemia include dysrhythmias (e.g., premature ventricular contractions), unexplained decreased level of consciousness (e.g., disorientation), dyspnea on exertion, and unexplained decreased urine output.

The patient had abdominal surgery yesterday. Today the lung sounds in the lower lobes have decreased. The nurse knows this could be due to what occurring? A Pain B Atelectasis C Pneumonia D Pleural effusion

B Rationale: Postoperatively there is an increased risk for atelectasis from anesthesia as well as restricted breathing from pain. Without deep breathing to stretch the alveoli, surfactant secretion to hold the alveoli open is not promoted. Pneumonia will occur later after surgery. Pleural effusion occurs because of blockage of lymphatic drainage or an imbalance between intravascular and oncotic fluid pressures, which is not expected in this case.

A nurse assesses a client who has a mediastinal chest tube. Which symptoms require the nurse's immediate intervention? (Select all that apply.) a. Production of pink sputum b. Tracheal deviation c. Pain at insertion site d. Sudden onset of shortness of breath e. Drainage greater than 70 mL/hr f. Disconnection at Y site

B, D, E, F

In assessment of the patient with acute respiratory distress, what should the nurse expect to observe (select all that apply)? A Cyanosis B Tripod position C Kussmaul respirations D Accessory muscle use E Increased AP diameter

B,D Rationale: Tripod position and accessory muscle use indicate moderate to severe respiratory distress. Cyanosis may be related to anemia, decreased oxygen transfer in the lungs, or decreased cardiac output. Therefore it is a nonspecific and unreliable indicator of only respiratory distress. Kussmaul respirations occur when the patient is in metabolic acidosis to increase CO2 excretion. Increased AP diameter occurs with lung hyperinflation from COPD, cystic fibrosis, or with advanced age.

During discharge teaching for a 65-year-old patient with COPD and pneumonia, which of the following vaccines should the nurse recommend that this patient receive? A) a. Staphylococcus aureus B) Haemophilus influenzae C) Pneumococcal D) Bacille-Calmette-Guérin (BCG)

C

The nurse evaluates that discharge teaching for a patient hospitalized with pneumonia has been most effective when the patient states which of the following measures to prevent a relapse? A) "I will seek immediate medical treatment for any upper respiratory infections." B) "I will increase my food intake to 2400 calories a day to keep my immune system well." C) "I should continue to do deep-breathing and coughing exercises for at least 6 weeks." D) "I must use home oxygen therapy for 3 months and then will have a chest x-ray to reevaluate."

C

The nurse is caring for a 73-year-old patient who underwent a left total knee arthroplasty. On the third postoperative day, the patient complains of shortness of breath, slight chest pain, and that "something is wrong." Temperature is 98.4o F, blood pressure 130/88, respirations 36, and oxygen saturation 91% on room air. Which of the following should the nurse first suspect as the etiology of this episode? A) New onset of angina pectoris B) Septic embolus from the knee joint C) Pulmonary embolus from deep vein thrombosis D) Pleural effusion related to positioning in the operating room

C

Which of the following clinical manifestations would the nurse expect to find during assessment of a patient admitted with pneumococcal pneumonia? A) Hyperresonance on percussion B) Vesicular breath sounds in all lobes C) Increased vocal fremitus on palpation D) Fine crackles in all lobes on auscultation

C

Which of the following is the priority nursing assessment in the care of a patient who has a tracheostomy? A) Electrolyte levels and daily weights B) Assessment of speech and swallowing C) Respiratory rate and oxygen saturation D) Pain assessment and assessment of mobility

C

Which of the following nursing interventions is most appropriate to enhance oxygenation in a patient with unilateral malignant lung disease? A) Positioning patient on right side B) Maintaining adequate fluid intake C) Positioning patient with "good lung down" D) Performing postural drainage every 4 hours

C

While ambulating a patient with metastatic lung cancer, the nurse observes a drop in oxygen saturation from 93% to 86%. Which of the following nursing interventions is most appropriate based upon these findings? A) Continue with ambulation since this is a normal response to activity. B) Obtain a physician's order for arterial blood gas determinations to verify the oxygen saturation. C) Obtain a physician's order for supplemental oxygen to be used during ambulation and other activity. D) Move the oximetry probe from the finger to the earlobe for more accurate monitoring during activity.

C

The nurse is interpreting a tuberculin skin test (TST) for a 58-year-old female patient with end-stage kidney disease secondary to diabetes mellitus. Which finding would indicate a positive reaction? A. Acid-fast bacilli cultured at the injection site B. 15-mm area of redness at the TST injection site C. 11-mm area of induration at the TST injection site D. Wheal formed immediately after intradermal injection

C Rationale: An area of induration ≥ 10 mm would be a positive reaction in a person with end-stage kidney disease. Reddened, flat areas do not indicate a positive reaction. A wheal appears when the TST is administered that indicates correct administration of the intradermal antigen. Presence of acid-fast bacilli in the sputum indicates active tuberculosis.

The patient is hospitalized with pneumonia. Which diagnostic test should be used to measure the efficiency of gas transfer in the lung and tissue oxygenation? A Thoracentesis B Bronchoscopy C Arterial blood gases D Pulmonary function tests

C Rationale: Arterial blood gases are used to assess the efficiency of gas transfer in the lung and tissue oxygenation as is pulse oximetry. Thoracentesis is used to obtain specimens for diagnostic evaluation, remove pleural fluid, or instill medication into the pleural space. Bronchoscopy is used for diagnostic purposes, to obtain biopsy specimens, and to assess changes resulting from treatment. Pulmonary function tests measure lung volumes and airflow to diagnose pulmonary disease, monitor disease progression, evaluate disability, and evaluate response to bronchodilators.

The nurse, when auscultating the lower lungs of the patient, hears these breath sounds. How should the nurse document these sounds? A Stridor B Rhonchi C Coarse crackles D Bronchovesicular

C Rationale: Coarse crackles are a series of long-duration, discontinuous, low-pitched sounds caused by air passing through an airway intermittently occluded by mucus, an unstable bronchial wall, or a fold of mucosa. Coarse crackles are evident on inspiration and at times expiration. Stridor is a continuous crowing sound of constant pitch from partial obstruction of larynx or trachea. Rhonchi are a continuous rumbling, snoring, or rattling sound from obstruction of large airways with secretions. Bronchovesicular sounds are normal sounds heard anteriorly over the mainstem bronchi on either side of the sternum and posteriorly between the scapulae with a medium pitch and intensity.

A 67-year-old male patient had a right total knee replacement 2 days ago. Upon auscultation of the patient's posterior chest, the nurse detects discontinuous, high-pitched breath sounds just before the end of inspiration in the lower portion of both lungs. Which statement most appropriately reflects how the nurse should document the breath sounds? A. "Bibasilar rhonchi present on inspiration." B. "Diminished breath sounds in the bases of both lungs." C. "Fine crackles posterior right and left lower lung fields." D. "Expiratory wheezing scattered throughout the lung fields."

C Rationale: Fine crackles are described as a series of short-duration, discontinuous, high-pitched sounds heard just before the end of inspiration.

A 71-year-old patient is admitted with acute respiratory distress related to cor pulmonale. Which of the following nursing interventions is most appropriate during admission of this patient? A) Perform a comprehensive health history with the patient to review prior respiratory problems. B) Complete a full physical examination to determine the effect of the respiratory distress on other body functions. C) Delay any physical assessment of the patient and review with the family the patient's history of respiratory problems. D) Perform a physical assessment of the respiratory system and ask specific questions related to this episode of respiratory distress.

D

The nurse is caring for a 73-year-old patient who underwent a left total knee arthroplasty. On the third postoperative day, the patient complains of shortness of breath, slight chest pain, and that "something is wrong." Temperature is 98.4o F, blood pressure 130/88, respirations 36, and oxygen saturation 91% on room air. Which of the following actions should the nurse take first? A) Notify the physician. B) Administer a nitroglycerine tablet sublingually. C) Conduct a thorough assessment of the chest pain. D) Sit the patient up in bed as tolerated and apply oxygen.

D

Which of the following is the priority nursing intervention in helping a patient expectorate thick lung secretions? A) Humidify the oxygen as able B) Administer cough suppressant q4hr C) Teach patient to splint the affected area D) Increase fluid intake to 3 L/day if tolerated

D Although several interventions may help the patient expectorate mucus, the highest priority should be on increasing fluid intake, which will liquefy the secretions so that the patient can expectorate them more easily. Humidifying the oxygen is also helpful, but is not the primary intervention. Teaching the patient to splint the affected area may also be helpful, but does not liquefy the secretions so that they can be removed.

What should the nurse inspect when assessing a patient with shortness of breath for evidence of long-standing hypoxemia? A. Chest excursion B. Spinal curvatures C. Respiratory pattern D. Fingernails and their base

D Rationale: Clubbing, a sign of long-standing hypoxemia, is evidenced by an increase in the angle between the base of the nail and the fingernail to 180 degrees or more, usually accompanied by an increase in the depth, bulk, and sponginess of the end of the finger.

A patient with a recent history of a dry cough has had a chest x-ray that revealed the presence of nodules. In an effort to determine whether the nodules are malignant or benign, what is the primary care provider likely to order? A Thoracentesis B Pulmonary angiogram C CT scan of the patient's chest D Positron emission tomography (PET)

D Rationale: PET is used to distinguish benign and malignant pulmonary nodules. Because malignant lung cells have an increased uptake of glucose, the PET scan (which uses an IV radioactive glucose preparation) can demonstrate increased uptake of glucose in malignant lung cells. This differentiation cannot be made using CT, a pulmonary angiogram, or thoracentesis.

After swallowing, a 73-year-old patient is coughing and has a wet voice. What changes of aging could be contributing to this abnormality? A Decreased response to hypercapnia B Decreased number of functional alveoli C Increased calcification of costal cartilage D Decreased respiratory defense mechanisms

D Rationale: These manifestations are associated with aspiration, which more easily occur in the right lung as the right mainstem bronchus is shorter, wider, and straighter than the left mainstem bronchus. Aspiration occurs more easily in the older patient related to decreased respiratory defense mechanisms (e.g., decreases in immunity, ciliary function, cough force, sensation in pharynx). Changes of aging include a decreased response to hypercapnia, decreased number of functional alveoli, and increased calcification of costal cartilage, but these do not increase the risk of aspiration.

When the patient is experiencing metabolic acidosis secondary to type 1 diabetes mellitus, what physiologic response should the nurse expect to assess in the patient? A Vomiting B Increased urination C Decreased heart rate D Rapid respiratory rate

D Rationale: When a patient with type 1 diabetes has hyperglycemia and ketonemia causing metabolic acidosis, the physiologic response is to increase the respiratory rate and tidal volume to blow off the excess CO2. Vomiting and increased urination may occur with hyperglycemia, but not as physiologic responses to metabolic acidosis. The heart rate will increase.

Home care for hospitalization with pneumonia

It is important for the patient to continue with coughing and deep breathing exercises for 6 to 8 weeks until all of the infection has cleared from the lungs. A patient should seek medical treatment for upper respiratory infections that persist for more than 7 days. Increased fluid intake, not caloric intake, is required to liquefy secretions. Home O2 is not a requirement unless the patient's oxygenation saturation is below normal.

A nurse auscultates a client's lung fields. Which action should the nurse take based on the lung sounds? (Click the media button to hear the audio clip.) a. Assess for airway obstruction. b. Initiate oxygen therapy. c. Assess vital signs. d. Elevate the client's head.

a. Assess for airway obstruction.

A nurse assesses a client who is prescribed fluticasone (Flovent) and notes oral lesions. Which action should the nurse take? a. Encourage oral rinsing after fluticasone administration. b. Obtain an oral specimen for culture and sensitivity. c. Start the client on a broad-spectrum antibiotic. d. Document the finding as a known side effect.

a. Encourage oral rinsing after fluticasone administration.

A nurse cares for a client with a 40-year smoking history who is experiencing distended neck veins and dependent edema. Which physiologic process should the nurse correlate with this client's history and clinical manifestations? a. Increased pulmonary pressure creating a higher workload on the right side of the heart b. Exposure to irritants resulting in increased inflammation of the bronchi and bronchioles c. Increased number and size of mucus glands producing large amounts of thick mucus d. Left ventricular hypertrophy creating a decrease in cardiac output

a. Increased pulmonary pressure creating a higher workload on the right side of the heart

A nurse auscultates a client's lung fields. Which pathophysiologic process should the nurse associate with this breath sound? (Click the media button to hear the audio clip.) a. Inflammation of the pleura b. Constriction of the bronchioles c. Upper airway obstruction d. Pulmonary vascular edema

a. Inflammation of the pleura

After teaching a client how to perform diaphragmatic breathing, the nurse assesses the client's understanding. Which action demonstrates that the client correctly understands the teaching? a. The client lays on his or her side with his or her knees bent. b. The client places his or her hands on his or her abdomen. c. The client lays in a prone position with his or her legs straight. d. The client places his or her hands above his or her head.

b. The client places his or her hands on his or her abdomen.

To prevent excessive pressure on tracheal capillaries, pressure in the cuff on a tracheostomy tube should be a. monitored every 2-3 days b. less than 20 mm Hg or 25 cm H2O c. less than 30 mm Hg or 35 cm H2O d. sufficient to fill the pilot balloon until it is tense

b higher pressures may compress tracheal capillaries, limit blood flow, and predispose to tracheal necrosis.

A patient is being discharged from the emergency department after being treated for epistaxis. In teaching the family first aid measures in the event the epistaxis would recur, which of the following measures would the nurse suggest (select all that apply)? A) Tilt patient's head backwards. B) Apply ice compresses to the nose. C) Tilt head forward while lying down. D) Pinch the entire soft lower portion of the nose. E) Partially insert a small gauze pad into the bleeding nostril.

b, d, e

A pulmonary nurse cares for clients who have chronic obstructive pulmonary disease (COPD). Which client should the nurse assess first? a. A 46-year-old with a 30-pack-year history of smoking b. A 52-year-old in a tripod position using accessory muscles to breathe c. A 68-year-old who has dependent edema and clubbed fingers d. A 74-year-old with a chronic cough and thick, tenacious secretions

b. A 52-year-old in a tripod position using accessory muscles to breathe

a type of tracheostomy tube that prevents speech is a. a cuffless tracheostomy tube b. a fenestrated tracheostomy tube c. a tube with an inflated foam cuff d. a cuffed tube with the cuff deflated

c

A patient is seen at the clinic with fever, muscle aches, sore throat with yellowish exudate, and headache. The nurse anticipates that the collaborative management will include (select all that apply) a. antiviral agents to treat influenza b. treatment with antibiotics starting ASAP c. a throat culture or rapid strep antigen test d. supportive care including cool, bland liquids e. comprehensive history to determine possible etiology

c, d, e

A nurse cares for a client with chronic obstructive pulmonary disease (COPD) who appears thin and disheveled. Which question should the nurse ask first? a. "Do you have a strong support system?" b. "What do you understand about your disease?" c. "Do you experience shortness of breath with basic activities?" d. "What medications are you prescribed to take each day?"

c. "Do you experience shortness of breath with basic activities?"

A nurse is teaching a client who has cystic fibrosis (CF). Which statement should the nurse include in this client's teaching? a. "Take an antibiotic each day." b. "Contact your provider to obtain genetic screening." c. "Eat a well-balanced, nutritious diet." d. "Plan to exercise for 30 minutes every day."

c. "Eat a well-balanced, nutritious diet."

After teaching a client who is prescribed a long-acting beta2 agonist medication, a nurse assesses the client's understanding. Which statement indicates the client comprehends the teaching? a. "I will carry this medication with me at all times in case I need it." b. "I will take this medication when I start to experience an asthma attack." c. "I will take this medication every morning to help prevent an acute attack." d. "I will be weaned off this medication when I no longer need it."

c. "I will take this medication every morning to help prevent an acute attack."

After teaching a client who is prescribed salmeterol (Serevent), the nurse assesses the client's understanding. Which statement by the client indicates a need for additional teaching? a. "I will be certain to shake the inhaler well before I use it." b. "It may take a while before I notice a change in my asthma." c. "I will use the drug when I have an asthma attack." d. "I will be careful not to let the drug escape out of my nose and mouth."

c. "I will use the drug when I have an asthma attack."

The nurse is caring for a client with lung cancer who states, "I don't want any pain medication because I am afraid I'll become addicted." How should the nurse respond? a. "I will ask the provider to change your medication to a drug that is less potent." b. "Would you like me to use music therapy to distract you from your pain?" c. "It is unlikely you will become addicted when taking medicine for pain." d. "Would you like me to give you acetaminophen (Tylenol) instead?"

c. "It is unlikely you will become addicted when taking medicine for pain."

A nurse cares for a client with chronic obstructive pulmonary disease (COPD). The client states that he no longer enjoys going out with his friends. How should the nurse respond? a. "There are a variety of support groups for people who have COPD." b. "I will ask your provider to prescribe you with an antianxiety agent." c. "Share any thoughts and feelings that cause you to limit social activities." d. "Friends can be a good support system for clients with chronic disorders."

c. "Share any thoughts and feelings that cause you to limit social activities."

A nurse cares for a female client who has a family history of cystic fibrosis. The client asks, "Will my children have cystic fibrosis?" How should the nurse respond? a. "Since many of your family members are carriers, your children will also be carriers of the gene." b. "Cystic fibrosis is an autosomal recessive disorder. If you are a carrier, your children will have the disorder." c. "Since you have a family history of cystic fibrosis, I would encourage you and your partner to be tested." d. "Cystic fibrosis is caused by a protein that controls the movement of chloride. Adjusting your diet will decrease the spread of this disorder."

c. "Since you have a family history of cystic fibrosis, I would encourage you and your partner to be tested."

A nurse cares for a client who tests positive for alpha1-antitrypsin (AAT) deficiency. The client asks, "What does this mean?" How should the nurse respond? a. "Your children will be at high risk for the development of chronic obstructive pulmonary disease." b. "I will contact a genetic counselor to discuss your condition." c. "Your risk for chronic obstructive pulmonary disease is higher, especially if you smoke." d. "This is a recessive gene and should have no impact on your health."

c. "Your risk for chronic obstructive pulmonary disease is higher, especially if you smoke."

The nurse instructs a client on how to correctly use an inhaler with a spacer. In which order should these steps occur? 1. "Press down firmly on the canister to release one dose of medication." 2. "Breathe in slowly and deeply." 3. "Shake the whole unit vigorously three or four times." 4. "Insert the mouthpiece of the inhaler into the nonmouthpiece end of the spacer." 5. "Place the mouthpiece into your mouth, over the tongue, and seal your lips tightly around the mouthpiece." 6. "Remove the mouthpiece from your mouth, keep your lips closed, and hold your breath for at least 10 seconds." a. 2, 3, 4, 5, 6, 1 b. 3, 4, 5, 1, 6, 2 c. 4, 3, 5, 1, 2, 6 d. 5, 3, 6, 1, 2, 4

c. 4, 3, 5, 1, 2, 6

A nurse cares for a client who is infected with Burkholderia cepacia. Which action should the nurse take first when admitting this client to a pulmonary care unit? a. Instruct the client to wash his or her hands after contact with other people. b. Implement Droplet Precautions and don a surgical mask. c. Keep the client isolated from other clients with cystic fibrosis. d. Obtain blood, sputum, and urine culture specimens.

c. Keep the client isolated from other clients with cystic fibrosis.

A nurse cares for a client who has developed esophagitis after undergoing radiation therapy for lung cancer. Which diet selection should the nurse provide for this client? a. Spaghetti with meat sauce, ice cream b. Chicken soup, grilled cheese sandwich c. Omelet, soft whole wheat bread d. Pasta salad, custard, orange juice

c. Omelet, soft whole wheat bread

A patient was seen in the clinic for an episode of epistaxis, which was controlled by placement of anterior nasal packing. During discharge teaching, the nurse instructs the patient to a. use asprin for pain relief b. remove the packing later that day c. skip the nex dose of antihypertensive medications d. avoid vigorous nose blowing and strenuous activity

d

When nursing a voice prosthesis, the patient a. swallows air using a Valsalva maneuver b. place a vibrating device in the mouth c. places a speaking valve over the stoma d. blocks the stoma entrance with a finger

d

Which of the following is a late symptom of head and neck cancer? a. hoarseness b. change in fit of dentures c. mouth ulcers that do not heal d. decreased mobility of the tongue

d

a patient with allergic rhinitis reports severe nasal congestion, sneezing, and watery, itchy eyes and nose at various times of the year. To teach the patient to control these symptoms, the nurse advises the patient to a. avoid all intranasal spray and oral antihystamines b. limit the duration of use of nasal decongestant spray for 10 days c. use oral decongestants at bedime to prevent symptoms during the night d. keep a diary of when the allergic reaction occurs and what precipitates it

d

while in the recover room, a patient with a total laryngectomy is suctioned and has blood mucus with some clots. Which of the following nursing interventions would apply? a. notify the physician immediately b. place the patient in the prone position to facilitate drainage c. install 3 mL of normal saline into the tracheotomy tube to loosen secretions d. continue your assessment of the patient, including O2 saturation, respiratory rate, and breath sounds

d

The nurse is caring for a client who is prescribed a long-acting beta2 agonist. The client states, "The medication is too expensive to use every day. I only use my inhaler when I have an attack." How should the nurse respond? a. "You are using the inhaler incorrectly. This medication should be taken daily." b. "If you decrease environmental stimuli, it will be okay for you to use the inhaler only for asthma attacks." c. "Tell me more about your fears related to feelings of breathlessness." d. "It is important to use this type of inhaler every day. Let's identify potential community services to help you."

d. "It is important to use this type of inhaler every day. Let's identify potential community services to help you."

A nurse assesses several clients who have a history of asthma. Which client should the nurse assess first? a. A 66-year-old client with a barrel chest and clubbed fingernails b. A 48-year-old client with an oxygen saturation level of 92% at rest c. A 35-year-old client who has a longer expiratory phase than inspiratory phase d. A 27-year-old client with a heart rate of 120 beats/min

d. A 27-year-old client with a heart rate of 120 beats/min

A nurse cares for a client who had a chest tube placed 6 hours ago and refuses to take deep breaths because of the pain. Which action should the nurse take? a. Ambulate the client in the hallway to promote deep breathing. b. Auscultate the client's anterior and posterior lung fields. c. Encourage the client to take shallow breaths to help with the pain. d. Administer pain medication and encourage the client to take deep breaths.

d. Administer pain medication and encourage the client to take deep breaths.

A nurse cares for a client who has a pleural chest tube. Which action should the nurse take to ensure safe use of this equipment? a. Strip the tubing to minimize clot formation and ensure patency. b. Secure tubing junctions with clamps to prevent accidental disconnections. c. Connect the chest tube to wall suction at the level prescribed by the provider. d. Keep padded clamps at the bedside for use if the drainage system is interrupted.

d. Keep padded clamps at the bedside for use if the drainage system is interrupted.

Therapeutic positioning for adequate breathing

identifies the best position for the patient assuring stable oxygenation status. Research indicates that positioning the patient with the unaffected lung (good lung) dependent best promotes oxygenation is patients with unilateral lung disease. For bilateral lung disease, the right lung down has best ventilation and perfusion. Increasing fluid intake and performing postural drainage will facilitate airway clearance, but positioning is most appropriate to enhance oxygenation.

To speak with a voice prosthesis

the patient manually blocks the stoma with the finger. Air moves from the lungs, through the prosthesis, into the esophagus, and out the mouth. Speech is produced by the air vibrating against the esophagus and is formed into words by moving the tongue and lips.


Related study sets

AP Biology- Ch. 3-5 Test(#26-50)

View Set

BADM 231 Midterm Practice Exam - Dr. Vincent van Empeh

View Set

Module 14. Differential Analysis

View Set